Search found 966 matches


Average of (20,60,100) = 180/3 = 60

Given 60 is 10 more than average of (40,80,x)

(or) Avg of (40,80,x) = 50

120+x = 3(50)

[spoiler]x = 30 (E)[/spoiler]

by shankar.ashwin

Thu May 31, 2012 9:08 am
Forum: Problem Solving
Topic: Average problem
Replies: 2
Views: 1311

kalifalk wrote:
The answer breaks down like this:

1.08c=1.5t or c/t=1.5/1.08.

in order to do percent change, however, wouldn't we need to do c + 1.08c = t + 1.5t?

I'm missing something and it's driving me nuts :)

Thanks!
A 8% increase signifies 1.08 times the value...

But, c + 1.08 = 2.08 = 108% increase

by shankar.ashwin

Wed Feb 22, 2012 2:23 am
Forum: Problem Solving
Topic: Ratios and Percents Question
Replies: 1
Views: 1465

Well this isnt exactly an alternate method... But then its a fast way using the same concept..

Set up the number similar to finding factors of 5..

5 | 200
5 | 40 - 0
5 | 8 - 0
5 | 1 - 3

Leave out the remainders.. You have 40+8+1 = 49.

by shankar.ashwin

Mon Feb 13, 2012 5:15 am
Forum: Problem Solving
Topic: Alternate Solutions Needed
Replies: 5
Views: 1808

BBR can be arranged in 3!/2! = 3 ways.

Now probability of getting BBR = (4/9)*(3/8)*(5/7) = 5/(3*14)

Now since we gotto consider various arrangements, multiply by 3..

Therefore, we have (5*3)/(3*14) = [spoiler]5/14[/spoiler]

by shankar.ashwin

Sun Feb 05, 2012 8:22 pm
Forum: Problem Solving
Topic: red&blue balls
Replies: 5
Views: 1613

pemdas wrote:yea, similar
updated the thread with my solution about the same http://www.beatthegmat.com/forming-team ... tml#448600 . Here -hint, Order MATTERS! hit it please
My first answer was a straight C, over thought it and messed it up :)..

Thanks Mitch and Pemdas

by shankar.ashwin

Fri Feb 03, 2012 11:22 am
Forum: Problem Solving
Topic: post mails
Replies: 13
Views: 2659

Mean of set = M

New mean will be = MN

% Increase = M(N-1)/M * 100 (or) (N-1)*100

we need to find N..

(1) doesnt give N
(2) doesnt give N

Together, we can find N.. C

by shankar.ashwin

Fri Feb 03, 2012 6:14 am
Forum: Data Sufficiency
Topic: Mean
Replies: 2
Views: 1072

Mean of 1,6,11,16,21 = 55/5 = 11

Now we are asked to find a set of numbers when added to this, the mean should not change
(or) in other words, the mean of the set added should also be = 11 (or) sum = 33.

Only III has sum = 33 - C

by shankar.ashwin

Fri Feb 03, 2012 6:08 am
Forum: Data Sufficiency
Topic: Mean 1
Replies: 1
Views: 1760

Mean of 2,8,10 and 12 = 32/4 = 8

25% increase = 1.25* 8 = 10

Now since 2 numbers are added, the sum of the 6 numbers should be 60

We know sum of given 4 = 32

Therefore, sum of N and K = 60-32 = 28

(N+K)^2 = 784 - D

by shankar.ashwin

Fri Feb 03, 2012 6:03 am
Forum: Data Sufficiency
Topic: Mean 2
Replies: 2
Views: 1774

1) m/n > 1

Both +ve - m>n
Both -ve - m<n

2) (m/n) - 1 > 1 - (n/m)

(m/n) + (n/m) > 2

say m =2 and n=1 (or) m=1 and n=2

Cant say if m>n

Together,

Again insufficient as both may -ve or +ve.. (say m=-2,n=-1) E

by shankar.ashwin

Fri Feb 03, 2012 5:59 am
Forum: Data Sufficiency
Topic: m>n?
Replies: 7
Views: 1849

Pemdas.. Just edited my post... Still dont get to any of the answer choices mentioned... Not sure whats wrong with the solution.. nice, i made this up not for enjoyment and easy answers :mrgreen: am i nasty? :twisted: @Shankar, yes Mitch has solved and briefly explained, but did you get there? it's ...

by shankar.ashwin

Fri Feb 03, 2012 5:52 am
Forum: Problem Solving
Topic: post mails
Replies: 13
Views: 2659

I am not sure of the formula... But this is similar to a problem in GMATPrep.. discussed several times here.. I think Mitch explains it well.. Have a look http://www.beatthegmat.com/forming-teams-t73034.html @shankar.ashwin do u mean we need to use this formula? The number of ways in which mn differ...

by shankar.ashwin

Fri Feb 03, 2012 2:54 am
Forum: Problem Solving
Topic: post mails
Replies: 13
Views: 2659

Assuming all 21 mails are distinct... We need to distribute them in 7 groups of 3. 21 different posts can be arranged in 21! ways.. Now since each courier receives 3 posts each, the order of the 3 doesnt matter.. For the 1st courier, we have (21*20*19)/3! For the 2nd courier, we have (18*17*16)/3! a...

by shankar.ashwin

Fri Feb 03, 2012 12:01 am
Forum: Problem Solving
Topic: post mails
Replies: 13
Views: 2659

Hi jeyachandar.. 0 is considered an even number.. So (1) will be sufficient as well.. (1) x and y are consecutive integers, one of them will be even.. hence xy is even (2) Given x/y is even, one of the numbers should be even.. hence product will be even too.. Can anyone help me with the question OG ...

by shankar.ashwin

Thu Feb 02, 2012 3:49 am
Forum: Data Sufficiency
Topic: Data sufficiency
Replies: 2
Views: 1088

harrybm wrote:
Maybe its a silly question, but why do you minus 1? what 1 represents?
You're asked range of positive integers in the set...

+ve integers are integers > 0.

1 is the lowest +ve integer and 7 is the greatest in the set..

Therefore, range = 7-1 = 6

by shankar.ashwin

Thu Feb 02, 2012 2:18 am
Forum: Problem Solving
Topic: Consecutive interger
Replies: 6
Views: 1764

Your list is correct.. However the range = |max - min| = |7 - (1)| = 6

Not sure why you subtracted 7 with 0.. 0 is not a positive integer..

Hope that helps.

by shankar.ashwin

Thu Feb 02, 2012 12:53 am
Forum: Problem Solving
Topic: Consecutive interger
Replies: 6
Views: 1764